LSAT and Law School Admissions Forum

Get expert LSAT preparation and law school admissions advice from PowerScore Test Preparation.

 Administrator
PowerScore Staff
  • PowerScore Staff
  • Posts: 8916
  • Joined: Feb 02, 2011
|
#26084
Complete Question Explanation
(See the complete passage discussion here: lsat/viewtopic.php?t=10837)

The correct answer choice is (A)

This question asks for the main point of the passage, so the answer is prephrased in the VIEWSTAMP discussion above: The author’s main point is that the paradox of omnipotence presents problems to sovereigns that can be addressed, if not entirely solved, with constitutional amendments

Answer choice (A): This is the correct answer choice, as it basically repeats the prephrase above: the paradox poses problems that are not necessarily solved with constitutional amendments.

Answer choice (B): The passage does not deal with “abstract theoretical paradoxes,” but rather focuses on one single specific paradox, providing concrete examples of the practical ramifications of the referenced paradox.

Answer choice (C): In the final paragraph, the author points out that issues persisted after the Glorious Revolution, not having been solved entirely but rather shifted from the Crown to the Parliament. This choice is not even accurate, so it cannot possibly provide the main point of the passage.


Answer choice (D): It is accurate that the Glorious Revolution did not provide a complete solution to the issue under discussion, but this is not the main point of the passage as discussed and prephrased above.

Answer choice (E): This choice is accurate, referencing the existence of a particular paradox, but it does not represent the author’s main point that omnipotence can be problematic for a sovereign, and constitutional amendments can address the issue but might not provide a complete solution.
 ylikate
  • Posts: 30
  • Joined: Aug 27, 2013
|
#11527
I chose D b/c I understood the passage as saying that the Glorious Revolution did not solve the problem, but merely shifted it from one branch (monach) to another (parliment). Please explain why this is incorrect?
 Steve Stein
PowerScore Staff
  • PowerScore Staff
  • Posts: 1153
  • Joined: Apr 11, 2011
|
#11554
Hi,

What you say is accurate, and it does pass the Fact Test, since this idea was explicitly mentioned in the last paragraph of the passage. However, this is not a Must Be True question, but a Main Point question, the answer to which should always be prephrased.

Did you prephrase the answer to this one? Looking back over the passage, how would you prephrase it now? What is this passage all about? The fact that one particular solution wasn't a real solution? Or the issue of having too much power, and being unable to place a limit on it?

Let me know your thoughts--thanks!

~Steve
 ylikate
  • Posts: 30
  • Joined: Aug 27, 2013
|
#11574
No, I didn't prephrase. I can see why A is more general and overarching than D. D has more of a "recency effect" on me since the Glorious Revolution and the issue with the monarchs are discussed towards the end of the passage.
 Steve Stein
PowerScore Staff
  • PowerScore Staff
  • Posts: 1153
  • Joined: Apr 11, 2011
|
#11582
Thanks for your response--that's great, because the issue is easily remedied. I've seen them use that trick successfully in other contexts as well--it's easy to be so impressed by accuracy that you forget, as you point out, to look for the overarching idea.

You know what to do! Force yourself to prephrase every time--particularly in Reading Comprehension, a section that features so many Must Be True and Main Point questions.

I hope that's helpful! It seems that this is pretty clear to you, but let me know--thanks!

~Steve
 Khodi7531
  • Posts: 116
  • Joined: Mar 14, 2018
|
#44607
So I was debating between A and D as well. Gut was leaning on A, but because I was going untimed I wanted to dig a little. I was able to knock D out because it says the Revolution "did not solve" the practical problem...when in fact it did - just not to the extent of completely abolishing it.

However, under time, even if my gut is pointing to A, how can I get D out of the way faster? I got rid of D after looking at it for like 5 minutes.

I guess with A I was thinking it also checked off two things: bringing up the omnipotence and its issues in general, and then about constitutional arrangements. While D was just saying, "it doesn't fully solve the issue" when in fact it does more than that.

How can I get faster at getting rid of those? This is my fight with RC. Very doable but I always need a lot of time for these 50/50s. Thanks!
 Shannon Parker
PowerScore Staff
  • PowerScore Staff
  • Posts: 147
  • Joined: Jun 08, 2016
|
#44688
Khodi7531 wrote:So I was debating between A and D as well. Gut was leaning on A, but because I was going untimed I wanted to dig a little. I was able to knock D out because it says the Revolution "did not solve" the practical problem...when in fact it did - just not to the extent of completely abolishing it.

However, under time, even if my gut is pointing to A, how can I get D out of the way faster? I got rid of D after looking at it for like 5 minutes.

I guess with A I was thinking it also checked off two things: bringing up the omnipotence and its issues in general, and then about constitutional arrangements. While D was just saying, "it doesn't fully solve the issue" when in fact it does more than that.

How can I get faster at getting rid of those? This is my fight with RC. Very doable but I always need a lot of time for these 50/50s. Thanks!
I see this a lot in MP questions. My first response is always, did you prephrase? Before you even read the question, did you have a sense in your mind of what the overall point of the entire passage was? This will help you get rid of attractive, but incorrect answer choices such as D. Answer choice D deals with a specific factual issue addressed in the passage, but does not reference the entire passage. The correct answer for a MP question will always encompass the overall point of the passage. Think of it as the why am I reading this question. What is it that the author wanted me to know?

Hope this clears it up.
Shannon

Get the most out of your LSAT Prep Plus subscription.

Analyze and track your performance with our Testing and Analytics Package.